Grandma is making a quilt. She has 540 cm of fabric to border the quilt. What is the greatest possible area for the quilt?
Question 1 options:


11 664 cm^2


18225 cm^2


72900 cm^2


291600 cm^2

Show your work:

Answers

Answer 1

Answer:

18225 cm²

Step-by-step explanation:

Divide 540 by 4 to get the length of all sides

540/4 = 135

Square 135 to get the max possible size

135² = 18225

Answer 2

18225 cm²  is the greatest possible area for the quilt.

What is area?

The measurement that expresses the size of a region on a plane or curved surface is called area. Surface area refers to the area of an open surface or the boundary of a three-dimensional object, whereas the area of a plane region or plane area refers to the area of a form or planar lamina.

Given

Divide 540 by 4 to obtain the length of all sides

540/4 = 135

Square 135 to acquire the max possible size

135² = 18225

18225 cm²  is the greatest possible area for the quilt.        

To learn more about area refer to:

https://brainly.com/question/25292087

#SPJ2


Related Questions

help lol i forgot everything of the summer time

fill in the table using this function rule

Answers

Answer:

hope it help you

Step-by-step explanation:

mark me brailiest answer

What I do is plug x into the equation to find y

1. Y=-4(-1)+3
Y=4+3
Y=7
2. Y=-4(0)+3
Y=0+3
Y=3
3. Y=-4(1)+3
Y=-4+3
Y=-1
4. Y=-4(2)+3
Y=-8+3
Y=-5

Hope this helps!! :)

4. a) x2 + xy2 + y 4

Answers

Answer:

x2 + xy2 + y 4=(x+y^2)^2-xy^2

Step-by-step explanation:

Helpppp pleaseeee !!!!!!

Answers

Answer:

149 inches squared

Step-by-step explanation:

top rectangle: 25 * 7 = 175

second rectangle: 8 * (25 - 17) = 8^2 = 64

triangle in bottom right: 1/2 * (13 - 8) * (15 - 11) = 10

175 + 64 + 10 = 149 sq in

hopefully got this right!

find the missing side. Round it the nearest tenth.​

Answers

Answer: x= 11√3= 19.0525 = 19.1

Step-by-step explanation:

Let the reference angle be 30

so

cos 30 = b/h

√3/2 = x/22

or, 22√3 = 2x

or. x = (22√3)/2

so, x = 11√3

Answer:

x = 19.1 cm

Step-by-step explanation:

→ Find the name of the side you are not given

Opposite

→ Find a formula without opposite in it

Cos = Adjacent ÷ Hypotenuse

→ Rearrange to make adjacent the subject

Adjacent = Cos × Hypotenuse

→ Substitute in the values

Adjacent = Cos ( 30 ) × 22

→ Simplify

Adjacent = 19.1

If give 7 billions for 7 millions people. What is total?

Answers

Answer:

this is a very big number

Step-by-step explanation:

(7000000000)⁷⁰⁰⁰⁰⁰⁰

Evaluate 4(3 - 1)^2..

Answers

Answer:

16

Step-by-step explanation:

4(3 - 1)^2

~Simplify using PEMDAS

4(2)^2

4(4)

16

Best of Luck!

Difference of the square of (x+6)(x+4)

Answers

Answer:

x^2 + 10x + 24

Step-by-step explanation:

(x+6)(x+4)

x^2 + 4x + 6x + 24

x^2 + 10x + 24

Amy, a nature photographer, randomly sampled photographs she took within the last year. She wanted to find out how many of her photographs contained flowers. The proportion of photographs that had flowers was 0.61, with a margin of error of 0.04. Construct a confidence interval for the proportion of her photographs taken within the last year contained flowers.

Answers

The Constructed  confidence interval for the proportion of her photographs taken within the last year contained flowers is

[tex]CI\ E(0.57,0.65)[/tex]

From the question we are told that:

The proportion of photographs that had flowers P= 0.61

Margin of error of M.E= 0.04

Generally, the equation for Confidence interval for proportion is mathematically given by

[tex]CI=0.61 \pm 0.04[/tex]P \pm M.E

Therefore Confidence interval is

[tex]CI=0.61 \pm 0.04[/tex]

And can also be written as

[tex]CI\ E((0.61+0.04),0.61-0.04))[/tex]

[tex]CI\ E(0.57,0.65)[/tex]

In conclusion the Confidence interval is

[tex]CI\ E(0.57,0.65)[/tex]

For more information on this visit

https://brainly.com/question/24131141?referrer=searchResults

Jay has a jar of beads. 6 purple beard, 8 blue beads, and 4 yellow beads. If he removes one bead at random, what is the probability that it will not be yellow?

Answers

You get a pie chart to do percentages just so you dont cheat you still can do work y'know

What would it be tho 300 doesn’t show up in my options my options are
1/49 -1/49 -49 and 49

Answers

1/49 would be the answer

SAT/ACT What is the solution of 1,200 – 5(3x + 30) = 600? A 30 B 50 C 150 D 200 E 250​

Answers

The answer you are looking for is letter A, x=30.

Solution/Explanation:

First, write out the equation,

1200-5(3x+30)=600

Next, using the Distributive Property,

1200-15x-150=600

Simplify the left side of the equation just a little bit more,

1050-15x=600

Reverse order of terms on the left side, to make it a little bit easier to solve,

-15x+1050=600

Now, subtract 1050 from both sides,

-15x+1050-1050=600-1050

Now, simplify this part of the equation,

-15x=-450

Finally, divide both sides by -15,

So, therefore, the final answer is x=30.

I hope this helped you. Enjoy your day, and take care!

I NEEDDD HELPPP ITSSSSSS URGENTTTTT!!!

Answers

Answer:

100 degrees

Step-by-step explanation:

measure of FHD = 65 + 35

measure of FHD = 100

Basically count/add up the total amount of degrees that are include in the angle <FHD.

-- (central angles)

So, 35 + 65 = 100 degrees

Find the length of x

Answers

Answer:

20

Step-by-step explanation:

Since the triangles are similar then the sides must be proportional

10/6 = x/12 cross multiply expressions

6x = 120 divide both sides by 6

x = 20

The system of linear equations x – y + 2 = 0 and 2x + y -14 = 0​

Answers

Answer:

(4, 6 )

Step-by-step explanation:

Given the equations

x - y + 2 = 0 ( subtract - y + 2 from both sides )

x = y - 2 → (1)

2x + y - 14 = 0 → (2)

Substitute x = y - 2 into (2)

2(y - 2) + y - 14 = 0 ← distribute and simplify left side

2y - 4 + y - 14 = 0

3y - 18 = 0 ( add 18 to both sides )

3y = 18 ( divide both sides by 3 )

y = 6

Substitute y = 6 into (1) for corresponding value of x

x = 6 - 2 = 4

solution is (4, 6 )

Given 12 consecutive integers, how many ways can three of these integers be selected to give a sum which divides by 4.

Disclaimer: A lot of points to be given, Full explanation required. Not only answer. Remember the sum of the number must be divisible by 4. I think modular arithmetic is the way to solve it, but who knows???

Answers

Answer:

55 ways

Step-by-step explanation:

Out of 12 consecutive integers:

3 - divide by 4, so the remainder is 0 3- give remainder of 1 3- give remainder of 2 3 - give remainder of 3

Sum of 3 integers will be divisible by 4 if the remainders are:

0 - 0 - 0 ⇒ 1 combination 0 - 1 - 3 ⇒ 3*3 = 9 combinations 0 - 3 - 1 ⇒ 3*3 = 9  combinations 1 - 1 - 2  ⇒ 2*3 = 6  combinations 1 - 2 - 1  ⇒ 2*3 = 6  combinations 2 - 1 - 1 ⇒ 2*3 = 6 combinations 3 - 0 - 1 ⇒ 3*3 = 9 combinations 3 - 1 - 0 ⇒ 3*3 = 9 combinations

So total number of combinations is:

1 + 9*4 + 6*3 = 55

Lilian is building a swimming pool in the shape of a right rectangular prism. The area of the base of the swimming pool is 72 square meters. The depth of the swimming pool is 3 meters. What is the volume of the swimming pool?

Answers

Answer:

216

Step-by-step explanation:

Volume of a rectangular prism = area of base * depth

Area of base: 72

Depth: 3

Volume = 72 * 3 = 216

What is an example in your house of a point ?

Answers

Answer:

Step-by-step explanation:

Nothing. A point is dimensionless. Ugh what an awful LOL.

How about the tip of a candle that has never been lit?

What is the smallest 3-digit palindrome that is divisible by both 3 and 4?

Answers

Answer:

252

Step-by-step explanation:

To be divisible by 3, it's digits have to add to a number that is a multiple of 3.

To be divisible by 4 its last 2 digits have to be divisible by 3.

So let's start with 1x1 which won't work because 1x1 is odd. so let's go to 2x2 and see what happens.

212 that's divisible by 4 but not 3

222 divisible by 3 but not 4

232 divisible by 4 but not 3

242 not divisible by either one.

252 I think this might be your answer

The digits add up to 9 which is a multiple of 3 and the last 2 digits are divisible by 4

2. Determine the measure of the angles indicated by letters. Justify your answers with the
properties or theorems you used.

Answers

Answer:

a = 50°

b = 130°

c = 50°

d = 50°

e = 130°

f = 130°

g = 50°

Answered by GAUTHMATH

PLS HELP ME ON THIS QUESTION I WILL MRK YOU AS BRAINLIEST IF YOU KNOW THE ANSWER!!
Which of the following measures is a measure of spread?
A. median
B. range
C. mode
D. mean

Answers

Answer:

range

Step-by-step explanation:

Answer:

B. range.

Step-by-step explanation:

others are:

» Standard variation.

» Interquatile range.

» Quatiles, deciles and percentiles.

» variance.

[tex]{ \underline{ \blue{ \sf{christ \: † \: alone}}}}[/tex]

3a + 2 = 20

5(b+1) = 10

3 (2y - 3) - 2y = y-3

2+ (2+4p) =6p

Please answer these questions with steps please!

Answers

1. 3a=20-2
3a=18
a=6

2. b+1=2
b=2-1
b=1

3. 6y-9-2y=y-3
4y-9=y-3
4y-y=-3+9
3y=6
y=2

4. 2+2+4p=6p
4+4p=6p
4p-6p=-4
-2p=-4
p=2

F is on the bisector of angle BCD. Find the length of FD (with lines over FD)

Answers

Answer:

8n-2 = 6n+9

2n-2 = 9

2n = 11

n = 5.5

So C is correct

Let me know if this helps!

how many inches is 775 centimeters

Answers

Answer:

305.11

Step-by-step explanation:

Just use a calculator. A centimeter is 2.5 inches. Divide 775 by that.

What is the area of a rectangle with vertices at (7,3) (12,3) (12,11) (7,11)

Answers

Answer:

Area = 5 × 8

= 40 square units

Answer:

40^2

Step by Step Solution:

I counted the difference between the length and the width, which was 5 and 8, then using the formula for area, lw=a^2, I did 5(8)=40^2.  Some people leave out the squared part of the area, but 40^2 would be the most correct option if they do not square any of the answers, just put 40 that'll probably be accepted too.

ax^2-y^2-x-y factorize​

Answers

Answer:

x(ax-1)-y(y+1)

Step-by-step explanation:

you have to group the like terms

ax^2-x-y^2-y

x(ax-1)-y(y+1)

I hope this helps

If LM = 9x + 27 and RS = 135, find x.

Answers

Answer:

x=12

Step-by-step explanation:

LM = RS

9x+27 = 135

Subtract 27 from each side

9x+27-27 =135-27

9x=108

Divide each side by 9

9x/9 = 108/9

x = 12

the boxes are equivalent so the one with a single dash is equal to the other with a single dash.

the one with 2 dashes is equal to the other with 2 dashes so on and so forth

SR=LM

LM=9x+27

RS=135

9x+27=135

so I solve it in my own weird way but you can solve it differently. 135-27=108

108/9=12

so your answer is 12

Two observers are 300 ft apart on opposite sides of a flagpole. The angles of
elevation from the observers to the top of the pole are 20°
and 15°. Find the
height of the flagpole.

Answers

I know similar questions and have answers. do you want

If a line has a midpoint at (2,5), and the endpoints are (0,0) and (4,y), what is the value of y? Please explain each step for a better understanding:)

Answers

Answer:

y = 10

Step-by-step explanation:

To find the y coordinate of the midpoint, take the y coordinates of the endpoints and average

(0+y)/2 = 5

Multiply each  die by 2

0+y = 10

y = 10

PLEASE HELP I WILL GIVE BRAINLIEST

Answers

Step-by-step explanation:

A natural number is a positive whole number.

A whole number is a positive number with no fractions or decimals.

A interger is a whole number negative or positive.

A rational number is a number that terminates or continue with repeating digits.

A irrational number is a number that doesn't terminate or continue with repeating digits.

1. Rational Number

2. Natural,Whole,Interger,Rational

3. Whole,Rational,Interger

4. Rational

5.Irrational

6.Rational

7.Natural,Whole,Interger,Rational

8.Interger,Rational

9.Irrational

Determine the sum of the first 33 terms of the following series:

−52+(−46)+(−40)+...

Answers

Answer:

1320

Step-by-step explanation:

Use the formula for sum of series, s(a) = n/2(2a + (n-1)d)

The terms increase by 6, so d is 6

a is the first term, -56

n is the terms you want to find, 33

Plug in the numbers, 33/2 (2(-56)+(32)6)

Simplify into 33(80)/2 and you get 1320

Other Questions
InstructionsCreate a multimedia project that contains the text element and all the contents that you have studied about that element Help pleaseee, Ill give brainly! Solve for p.19p2p+16p+12=18p= in 16 years, Marissa will be five times older than she is today. How old is she? The Zeller Corporation's stockholders' equity accounts have the following balances as of December 31, 2016:Common stock, $10 par (30,000 shares issued and outstanding) $300,000Additional paid-in capital 2,000,000Retained earnings 5,700,000Total stockholders' equity $8,000,000Refer to above table. On January 2, 2017, the board of directors of Zeller declared a 5% stock dividend to be distributed on January 31, 2017. The market price per share of Zeller's common stock was $30 on January 2 and $32 on January 31. As a result of this stock dividend, the retained earnings account should be decreased by :___________ What is | 34 | ?-------l l-------l l-------- Determine what type of model best fits the given situation:A. linearB. exponentialO c. quadraticD. none of theseReset Selection Damaged items are marked down 25% to 40%. A newspaper coupon holder; to an additional 10% markdown of the new price due to the damage. What is the lowest price of a damaged item that was originally marked GHC100? A GHC 35.60 B GH 5000 C. GHC 18.00 D. GHC 40.80 E GHC 23.40 An individual taxpayer reports the following items for the current year: Ordinary income from Partnership A, operating a movie theater in which the taxpayer materially participates $70,000 Net loss from Partnership B, operating an equipment rental business in which the taxpayer does not materially participate (9,000) Rental income from building rented to a third party 7,000 Short-term capital gain from sale of stock 4,000 What is the taxpayers adjusted gross income for the year? NEED HALP!!! Find the ordered pair $(s,t)$ that satisfies the system The midpoint of has coordinates of (4, -9). The endpoint A has coordinates (-3, -5). What are the coordinates of B? The majority of immigrants that entered the United States during the industrial age came from Most of the heat loss for outdoor swimming pools is due to surfaceevaporation. So, the greater the area of the surface of the pool, the greaterthe heat loss. For a given perimeter, which surface shape would be moreefficient at retaining heat: a circle or a rectangle? Justify your answer. Please whoever knows they can solve this, help!I have attached the question down below. Please explain in detail and diagrams if the question permits you to do so. Thank you. I will also be marking the brainiest. Write 5 examples of Declarative sentences, exclamatory, interrogative, and imperative sentences. Which of the following statements is true?a. Overhead can be applied slowly as a job is worked on.b. Overhead can be applied when the job is completed.c. Overhead should be applied to any job not completed at year-end in order to properly value the work in process inventory. A project has an expected risky cash flow of $500 in year 3. The risk-free rate is 4%, the expected market rate of return is 14%, and the project's beta is 1.20. Calculate the certainty equivalent cash flow for year 3, CEQ3. (Assume CAPM holds.) Explain why they substituted cos(60) with 1/2 ? (Look at image) Quy tc suy lun no l c s ca suy din sau: "Bit rng: 2 ng thng d v d' song song hoc ct nhau. Ta c d khng song song vi d'. Vy d ct d'Select one:a. Lut loi trb. Lut rt gnc. Lut phn chngd. Lut tch ri An object travels along the x-axis so that its position after t seconds is given by x(t) = 2t2 5t 18 for all times t such that t 0.Which inequality describes all times t for which the object is traveling toward the right?